Difference between revisions of "1999 AIME Problems/Problem 14"

(solutions by (1) joml88 (2) 1234567890)
m (Solution 4)
(12 intermediate revisions by 9 users not shown)
Line 1: Line 1:
 
== Problem ==
 
== Problem ==
Point <math>P_{}</math> is located inside traingle <math>ABC</math> so that angles <math>PAB, PBC,</math> and <math>PCA</math> are all congruent.  The sides of the triangle have lengths <math>AB=13, BC=14,</math> and <math>CA=15,</math> and the tangent of angle <math>PAB</math> is <math>m/n,</math> where <math>m_{}</math> and <math>n_{}</math> are relatively prime positive integers.  Find <math>m+n.</math>
+
[[Point]] <math>P_{}</math> is located inside [[triangle]] <math>ABC</math> so that [[angle]]s <math>PAB, PBC,</math> and <math>PCA</math> are all congruent.  The sides of the triangle have lengths <math>AB=13, BC=14,</math> and <math>CA=15,</math> and the [[tangent]] of angle <math>PAB</math> is <math>m/n,</math> where <math>m_{}</math> and <math>n_{}</math> are relatively [[prime]] positive integers.  Find <math>m+n.</math>
  
 
__TOC__
 
__TOC__
 
== Solution ==
 
== Solution ==
[[Image:1999_AIME-14.png]]
+
<center><asy>
 +
real theta = 29.66115; /* arctan(168/295) to five decimal places .. don't know other ways to construct Brocard */
 +
pathpen = black +linewidth(0.65); pointpen = black;
 +
pair A=(0,0),B=(13,0),C=IP(circle(A,15),circle(B,14));
 +
D(MP("A",A)--MP("B",B)--MP("C",C,N)--cycle);
 +
 
 +
/* constructing P, C is there as check */
 +
pair Aa=A+(B-A)*dir(theta),Ba=B+(C-B)*dir(theta),Ca=C+(A-C)*dir(theta), P=IP(A--Aa,B--Ba);
 +
D(A--MP("P",P,NW)--B);D(P--C);
 +
D(anglemark(B,A,P,30));D(anglemark(C,B,P,30));D(anglemark(A,C,P,30));
 +
MP("13",(A+B)/2,S);MP("15",(A+C)/2,NW);MP("14",(C+B)/2,NE);
 +
</asy></center>
 +
<!--This image does exist now: [[Image:1999_AIME-14.png]]-->
 +
 
 
=== Solution 1 ===
 
=== Solution 1 ===
Drop perpendiculars from <math>P</math> to the three sides of <math>\triangle ABC</math> and let them meet <math>\overline{AB}, \overline{BC},</math> and <math>\overline{CA}</math> at <math>D, E,</math> and <math>F</math> respectively.  Let <math>BE = x, CF = y,</math> and <math>AD = z</math>.  We have that
+
Drop [[perpendicular]]s from <math>P</math> to the three sides of <math>\triangle ABC</math> and let them meet <math>\overline{AB}, \overline{BC},</math> and <math>\overline{CA}</math> at <math>D, E,</math> and <math>F</math> respectively.   
 +
 
 +
<center><asy>
 +
import olympiad;
 +
real theta = 29.66115; /* arctan(168/295) to five decimal places .. don't know other ways to construct Brocard */
 +
pathpen = black +linewidth(0.65); pointpen = black;
 +
pair A=(0,0),B=(13,0),C=IP(circle(A,15),circle(B,14));
 +
D(MP("A",A)--MP("B",B)--MP("C",C,N)--cycle);
 +
 
 +
/* constructing P, C is there as check */
 +
pair Aa=A+(B-A)*dir(theta),Ba=B+(C-B)*dir(theta),Ca=C+(A-C)*dir(theta), P=IP(A--Aa,B--Ba);
 +
D(A--MP("P",P,SSW)--B);D(P--C);
 +
D(anglemark(B,A,P,30));D(anglemark(C,B,P,30));D(anglemark(A,C,P,30));
 +
MP("13",(A+B)/2,S);MP("15",(A+C)/2,NW);MP("14",(C+B)/2,NE);
 +
 
 +
/* constructing D,E,F as foot of perps from P */
 +
pair D=foot(P,A,B),E=foot(P,B,C),F=foot(P,C,A);
 +
D(MP("D",D,NE)--P--MP("E",E,SSW),dashed);D(P--MP("F",F),dashed);
 +
D(rightanglemark(P,E,C,15));D(rightanglemark(P,F,C,15));D(rightanglemark(P,D,A,15));
 +
</asy></center>
 +
 
 +
Let <math>BE = x, CF = y,</math> and <math>AD = z</math>.  We have that
 
<cmath>
 
<cmath>
\begin{eqnarray*} DP & = & z\tan \theta \\
+
\begin{align*}DP&=z\tan\theta\\
EP & = & x\tan \theta \\
+
EP&=x\tan\theta\\
FP & = & y\tan \theta \end{eqnarray*}
+
FP&=y\tan\theta\end{align*}
 
</cmath>
 
</cmath>
 
We can then use the tool of calculating area in two ways
 
We can then use the tool of calculating area in two ways
 
<cmath>
 
<cmath>
\begin{eqnarray*} [ABC] & = & [PAB] + [PBC] + [PCA] \\
+
\begin{align*}[ABC]&=[PAB]+[PBC]+[PCA]\\
& = & \frac 12 (13)(z\tan \theta) + \frac 12 (14)(x\tan\theta) + \frac 12 (15)(y\tan\theta) \\
+
&=\frac{1}{2}(13)(z\tan\theta)+\frac{1}{2}(14)(x\tan\theta)+\frac{1}{2}(15)(y\tan\theta)\\
& = & \frac 12 \tan\theta(13z + 14x + 15y) \end{eqnarray*}
+
&=\frac{1}{2}\tan\theta(13z+14x+15y)\end{align*}
 
</cmath>
 
</cmath>
On the other hand
+
On the other hand,
 
<cmath>
 
<cmath>
\begin{eqnarray*} [ABC] & = & \sqrt {s(s - a)(s - b)(s - c)} \\
+
\begin{align*}[ABC]&=\sqrt{s(s-a)(s-b)(s-c)}\\
& = & \sqrt {21\cdot 6\cdot 7\cdot 8} \\
+
&=\sqrt{21\cdot6\cdot7\cdot8}\\
& = & 84 \end{eqnarray*}
+
&=84\end{align*}
 
</cmath>
 
</cmath>
We still need <math>13z + 14x + 15y</math> though. We have all these right triangles and we haven't even touched Pythagoras. So we give it a shot
+
We still need <math>13z+14x+15y</math> though. We have all these [[right triangle]]s and we haven't even touched [[Pythagorean theorem|Pythagoras]]. So we give it a shot:
 
<cmath>
 
<cmath>
\begin{eqnarray} x^2 + x^2\tan^2\theta & = & z^2\tan^2\theta + (13 - z)^2 \\
+
\begin{align}x^2+x^2\tan^2\theta&=z^2\tan^2\theta+(13-z)^2\\
z^2 + z^2\tan^2\theta & = & y^2\tan^2\theta + (15 - y)^2 \\
+
z^2+z^2\tan^2\theta&=y^2\tan^2\theta+(15-y)^2\\
y^2 + y^2\tan^2\theta & = & x^2\tan^2\theta + (14 - x)^2 \\
+
y^2+y^2\tan^2\theta&=x^2\tan^2\theta+(14-x)^2\end{align}
\end{eqnarray*}
 
 
</cmath>
 
</cmath>
But then <math>(1) + (2) + (3)</math> gives
+
Adding <math>(1) + (2) + (3)</math> gives
 
<cmath>
 
<cmath>
\begin{eqnarray*} x^2 + y^2 + z^2 & = & (14 - x)^2 + (15 - y)^2 + (13 - z)^2 \\
+
\begin{align*}x^2+y^2+z^2&=(14-x)^2+(15-y)^2+(13-z)^2\\
\Rightarrow 13z + 14x + 15y & = & 295 \end{eqnarray*}
+
\Rightarrow13z+14x+15y&=295\end{align*}
 
</cmath>
 
</cmath>
Recall that we found that <math>[ABC] = \frac 12 \tan\theta(13z + 14x + 15y) = 84</math>. Plugging in <math>13z + 14x + 15y = 295</math> we get <math>\tan \theta = \frac {168}{295}</math> giving us <math>\boxed{463}</math> for an answer.
+
Recall that we found that <math>[ABC]=\frac{1}{2}\tan\theta(13z+14x+15y)=84</math>. Plugging in <math>13z+14x+15y=295</math>, we get <math>\tan\theta=\frac{168}{295}</math>, giving us <math>\boxed{463}</math> for an answer.
  
 
=== Solution 2 ===
 
=== Solution 2 ===
Let <math>AB = c, BC = a, AC = b, PA = x, PB = y, PC = z</math>.
+
Let <math>AB=c</math>, <math>BC=a</math>, <math>AC=b</math>, <math>PA=x</math>, <math>PB=y</math>, and <math>PC=z</math>.
 
 
So by the Law of Cosines, we have:
 
<math>x^2 = z^2 + b^2 - 2bz\cos{\theta}</math>
 
<math>y^2 = x^2 + c^2 - 2cx\cos{\theta}</math>
 
<math>z^2 = y^2 + a^2 - 2ay\cos{\theta}</math>
 
  
 +
So by the [[Law of Cosines]], we have:
 +
<cmath>
 +
\begin{align*}x^2 &= z^2 + b^2 - 2bz\cos{\theta}\\
 +
y^2 &= x^2 + c^2 - 2cx\cos{\theta}\\
 +
z^2 &= y^2 + a^2 - 2ay\cos{\theta}\end{align*}
 +
</cmath>
 
Adding these equations and rearranging, we have:
 
Adding these equations and rearranging, we have:
<math>a^2 + b^2 + c^2 = (2bz + 2cx + 2ay)\cos{\theta}</math>                (1)
+
<cmath>
 
+
a^2 + b^2 + c^2 = (2bz + 2cx + 2ay)\cos{\theta}\qquad(1)
Now <math>[CAP] + [ABP] + [BCP] = [ABC] = \sqrt {(21)(8)(7)(6)} = 84</math>, by Heron's Formula.
+
</cmath>
 +
Now <math>[CAP] + [ABP] + [BCP] = [ABC] = \sqrt {(21)(8)(7)(6)} = 84</math>, by [[Heron's formula]].
  
 
Now the area of a triangle, <math>[A] = \frac {mn\sin{\beta}}{2}</math>, where <math>m</math> and <math>n</math> are sides on either side of an angle, <math>\beta</math>. So,  
 
Now the area of a triangle, <math>[A] = \frac {mn\sin{\beta}}{2}</math>, where <math>m</math> and <math>n</math> are sides on either side of an angle, <math>\beta</math>. So,  
<math>[CAP] = \frac {bz\sin{\theta}}{2}</math>.
+
<cmath>
<math>[ABP] = \frac {cx\sin{\theta}}{2}</math>.
+
\begin{align*}[CAP] &= \frac {bz\sin{\theta}}{2}\\
<math>[BCP] = \frac {ay\sin{\theta}}{2}</math>
+
[ABP] &= \frac {cx\sin{\theta}}{2}\\
 +
[BCP] &= \frac {ay\sin{\theta}}{2}\end{align*}
 +
</cmath>
 +
Adding these equations yields:
 +
<cmath>\begin{align*}[ABC]= 84 &= \frac {(bz + cx + ay)\sin{\theta}}{2}\\
 +
\Rightarrow 168&= (bz + cx + ay)\sin{\theta}\qquad (2)\end{align*}
 +
</cmath>
 +
Dividing <math>(2)</math> by <math>(1)</math>, we have:
 +
<cmath>
 +
\begin{align*}\frac {168}{a^2 + b^2 + c^2} &= \frac {(bz + cx + ay)\sin{\theta}}{(2bz + 2cx + 2ay)\cos{\theta}}\\
 +
\Rightarrow \tan{\theta} = \frac {336}{a^2 + b^2 + c^2} &= \frac {336}{14^2 + 15^2 + 13^2} = \frac {336}{590} = \frac {168}{295}\end{align*}
 +
</cmath>
 +
Thus, <math>m + n = 168 + 295 = \boxed{463}</math>.
  
So adding these equations yields:
+
Note: In fact, this problem is unfairly easy to those who happen to have learned about Brocard point. The Brocard Angle is given by
<math>[ABC] = 84 = \frac {(bz + cx + ay)\sin{\theta}}{2}</math>
+
<cmath>cot(\theta)=\frac{a^2+b^2+c^2}{4\Delta}</cmath>
<math>\Rightarrow 168 = (bz + cx + ay)\sin{\theta}</math>                                       (2)
 
  
Dividing (2) by (1), we have:
+
=== Solution 3 ===
<math>\frac {168}{a^2 + b^2 + c^2} = \frac {(bz + cx + ay)\sin{\theta}}{(2bz + 2cx + 2ay)\cos{\theta}}</math>
 
<math>\Rightarrow \tan{\theta} = \frac {336}{a^2 + b^2 + c^2} = \frac {336}{14^2 + 15^2 + 13^2} = \frac {336}{590} = \frac {168}{295}</math>
 
  
So <math>m + n = 168 + 295 = \boxed{463}</math>.
+
Let <math>\angle{PAB} = \angle{PBC} = \angle{PCA} = x.</math> Then, using Law of Cosines on the three triangles containing vertex <math>P,</math> we have
 +
<cmath>
 +
\begin{align*}
 +
b^2 &= a^2 + 169 - 26a \cos x \\
 +
c^2 &= b^2 + 196 - 28b \cos x \\
 +
a^2 &= c^2 + 225 - 30c \cos x.
 +
\end{align*}
 +
</cmath>
 +
Add the three equations up and rearrange to obtain <cmath>(13a + 14b + 15c) \cos x = 295.</cmath> Also, using <math>[ABC] = \frac{1}{2}ab \sin \angle C</math> we have <cmath>[ABC] = [APB] + [BPC] + [CPA] = \dfrac{\sin x}{2}(13a + 14b + 15c) = 84 \iff (13a + 14b + 15c) \sin x = 168.</cmath> Divide the two equations to obtain <math>\tan x = \frac{168}{295} \iff \boxed{463}.~\square</math>
  
 
== See also ==
 
== See also ==
Line 71: Line 123:
  
 
[[Category:Intermediate Geometry Problems]]
 
[[Category:Intermediate Geometry Problems]]
 +
{{MAA Notice}}

Revision as of 22:31, 28 May 2021

Problem

Point $P_{}$ is located inside triangle $ABC$ so that angles $PAB, PBC,$ and $PCA$ are all congruent. The sides of the triangle have lengths $AB=13, BC=14,$ and $CA=15,$ and the tangent of angle $PAB$ is $m/n,$ where $m_{}$ and $n_{}$ are relatively prime positive integers. Find $m+n.$

Solution

[asy] real theta = 29.66115; /* arctan(168/295) to five decimal places .. don't know other ways to construct Brocard */  pathpen = black +linewidth(0.65); pointpen = black; pair A=(0,0),B=(13,0),C=IP(circle(A,15),circle(B,14)); D(MP("A",A)--MP("B",B)--MP("C",C,N)--cycle);  /* constructing P, C is there as check */ pair Aa=A+(B-A)*dir(theta),Ba=B+(C-B)*dir(theta),Ca=C+(A-C)*dir(theta), P=IP(A--Aa,B--Ba);  D(A--MP("P",P,NW)--B);D(P--C); D(anglemark(B,A,P,30));D(anglemark(C,B,P,30));D(anglemark(A,C,P,30)); MP("13",(A+B)/2,S);MP("15",(A+C)/2,NW);MP("14",(C+B)/2,NE); [/asy]

Solution 1

Drop perpendiculars from $P$ to the three sides of $\triangle ABC$ and let them meet $\overline{AB}, \overline{BC},$ and $\overline{CA}$ at $D, E,$ and $F$ respectively.

[asy] import olympiad; real theta = 29.66115; /* arctan(168/295) to five decimal places .. don't know other ways to construct Brocard */  pathpen = black +linewidth(0.65); pointpen = black; pair A=(0,0),B=(13,0),C=IP(circle(A,15),circle(B,14)); D(MP("A",A)--MP("B",B)--MP("C",C,N)--cycle);  /* constructing P, C is there as check */ pair Aa=A+(B-A)*dir(theta),Ba=B+(C-B)*dir(theta),Ca=C+(A-C)*dir(theta), P=IP(A--Aa,B--Ba);  D(A--MP("P",P,SSW)--B);D(P--C); D(anglemark(B,A,P,30));D(anglemark(C,B,P,30));D(anglemark(A,C,P,30)); MP("13",(A+B)/2,S);MP("15",(A+C)/2,NW);MP("14",(C+B)/2,NE);  /* constructing D,E,F as foot of perps from P */ pair D=foot(P,A,B),E=foot(P,B,C),F=foot(P,C,A); D(MP("D",D,NE)--P--MP("E",E,SSW),dashed);D(P--MP("F",F),dashed); D(rightanglemark(P,E,C,15));D(rightanglemark(P,F,C,15));D(rightanglemark(P,D,A,15)); [/asy]

Let $BE = x, CF = y,$ and $AD = z$. We have that \begin{align*}DP&=z\tan\theta\\ EP&=x\tan\theta\\ FP&=y\tan\theta\end{align*} We can then use the tool of calculating area in two ways \begin{align*}[ABC]&=[PAB]+[PBC]+[PCA]\\ &=\frac{1}{2}(13)(z\tan\theta)+\frac{1}{2}(14)(x\tan\theta)+\frac{1}{2}(15)(y\tan\theta)\\ &=\frac{1}{2}\tan\theta(13z+14x+15y)\end{align*} On the other hand, \begin{align*}[ABC]&=\sqrt{s(s-a)(s-b)(s-c)}\\ &=\sqrt{21\cdot6\cdot7\cdot8}\\ &=84\end{align*} We still need $13z+14x+15y$ though. We have all these right triangles and we haven't even touched Pythagoras. So we give it a shot: \begin{align}x^2+x^2\tan^2\theta&=z^2\tan^2\theta+(13-z)^2\\ z^2+z^2\tan^2\theta&=y^2\tan^2\theta+(15-y)^2\\ y^2+y^2\tan^2\theta&=x^2\tan^2\theta+(14-x)^2\end{align} Adding $(1) + (2) + (3)$ gives \begin{align*}x^2+y^2+z^2&=(14-x)^2+(15-y)^2+(13-z)^2\\ \Rightarrow13z+14x+15y&=295\end{align*} Recall that we found that $[ABC]=\frac{1}{2}\tan\theta(13z+14x+15y)=84$. Plugging in $13z+14x+15y=295$, we get $\tan\theta=\frac{168}{295}$, giving us $\boxed{463}$ for an answer.

Solution 2

Let $AB=c$, $BC=a$, $AC=b$, $PA=x$, $PB=y$, and $PC=z$.

So by the Law of Cosines, we have: \begin{align*}x^2 &= z^2 + b^2 - 2bz\cos{\theta}\\ y^2 &= x^2 + c^2 - 2cx\cos{\theta}\\ z^2 &= y^2 + a^2 - 2ay\cos{\theta}\end{align*} Adding these equations and rearranging, we have: \[a^2 + b^2 + c^2 = (2bz + 2cx + 2ay)\cos{\theta}\qquad(1)\] Now $[CAP] + [ABP] + [BCP] = [ABC] = \sqrt {(21)(8)(7)(6)} = 84$, by Heron's formula.

Now the area of a triangle, $[A] = \frac {mn\sin{\beta}}{2}$, where $m$ and $n$ are sides on either side of an angle, $\beta$. So, \begin{align*}[CAP] &= \frac {bz\sin{\theta}}{2}\\ [ABP] &= \frac {cx\sin{\theta}}{2}\\ [BCP] &= \frac {ay\sin{\theta}}{2}\end{align*} Adding these equations yields: \begin{align*}[ABC]= 84 &= \frac {(bz + cx + ay)\sin{\theta}}{2}\\ \Rightarrow 168&= (bz + cx + ay)\sin{\theta}\qquad (2)\end{align*} Dividing $(2)$ by $(1)$, we have: \begin{align*}\frac {168}{a^2 + b^2 + c^2} &= \frac {(bz + cx + ay)\sin{\theta}}{(2bz + 2cx + 2ay)\cos{\theta}}\\ \Rightarrow \tan{\theta} = \frac {336}{a^2 + b^2 + c^2} &= \frac {336}{14^2 + 15^2 + 13^2} = \frac {336}{590} = \frac {168}{295}\end{align*} Thus, $m + n = 168 + 295 = \boxed{463}$.

Note: In fact, this problem is unfairly easy to those who happen to have learned about Brocard point. The Brocard Angle is given by \[cot(\theta)=\frac{a^2+b^2+c^2}{4\Delta}\]

Solution 3

Let $\angle{PAB} = \angle{PBC} = \angle{PCA} = x.$ Then, using Law of Cosines on the three triangles containing vertex $P,$ we have \begin{align*} b^2 &= a^2 + 169 - 26a \cos x \\ c^2 &= b^2 + 196 - 28b \cos x \\ a^2 &= c^2 + 225 - 30c \cos x. \end{align*} Add the three equations up and rearrange to obtain \[(13a + 14b + 15c) \cos x = 295.\] Also, using $[ABC] = \frac{1}{2}ab \sin \angle C$ we have \[[ABC] = [APB] + [BPC] + [CPA] = \dfrac{\sin x}{2}(13a + 14b + 15c) = 84 \iff (13a + 14b + 15c) \sin x = 168.\] Divide the two equations to obtain $\tan x = \frac{168}{295} \iff \boxed{463}.~\square$

See also

1999 AIME (ProblemsAnswer KeyResources)
Preceded by
Problem 13
Followed by
Problem 15
1 2 3 4 5 6 7 8 9 10 11 12 13 14 15
All AIME Problems and Solutions

The problems on this page are copyrighted by the Mathematical Association of America's American Mathematics Competitions. AMC logo.png